GATE 2021 Humanities and Social Sciences (XH), Psychology (XH-C5) Question Paper Available - Download Here with Solution PDF

Shivam Yadav's profile photo

Shivam Yadav

Updated on - Jan 13, 2026

GATE 2021 Humanities and Social Sciences (XH), Psychology (XH-C5) Question Paper with Solutions is available now. GATE 2021 XH was conducted on 14th February by IIT Bombay. GATE 2021 XH comprises several optional subjects sections out of which one section is compulsory to attempt. The total marks of the exam was 100. The overall difficulty level of GATE 2021 XH-C5 was easy to moderate. Candidates would get 3 hours to solve the questions listed in the two sections of GATE 2021 XH.

GATE 2021 Humanities and Social Sciences (XH), Psychology (XH-C5) Question Paper with Solutions

GATE 2021 Humanities and Social Sciences (XH), Psychology (XH-C5) Question Paper download iconDownload Check Solutions
GATE 2021 Humanities and Social Sciences (XH), Psychology (XH-C5) Question Paper with Solutions



Question 1:

Consider the following sentences:
(i) After his surgery, Raja hardly could walk.
(ii) After his surgery, Raja could barely walk.
(iii) After his surgery, Raja barely could walk.
(iv) After his surgery, Raja could hardly walk.
Which of the above sentences are grammatically CORRECT?

  • (A) (i) and (ii)
  • (B) (i) and (iii)
  • (C) (iii) and (iv)
  • (D) (ii) and (iv)
Correct Answer: (D) (ii) and (iv)
View Solution




Let's examine each sentence in detail:

Sentence (i): "After his surgery, Raja hardly could walk."
- This sentence is incorrect. In English, when we use the adverb hardly, it should appear before the verb in most cases. The correct structure would be: "Raja could hardly walk." Therefore, the sentence is grammatically wrong because the placement of "hardly" is incorrect.

Sentence (ii): "After his surgery, Raja could barely walk."
- This sentence is correct. The adverb barely is placed before the verb (as it should be), and the sentence follows the proper English structure. The meaning of the sentence is clear, and it is grammatically sound.

Sentence (iii): "After his surgery, Raja barely could walk."
- This sentence is incorrect. Although the adverb barely is used correctly, the position of the adverb is wrong. In standard English usage, barely should come before the verb "could," not after it. The correct sentence would be: "Raja could barely walk."

Sentence (iv): "After his surgery, Raja could hardly walk."
- This sentence is correct. The adverb hardly is placed correctly before the verb "could," making the sentence grammatically correct. This sentence properly conveys that Raja had difficulty walking after his surgery.

Thus, the sentences (ii) and (iv) are grammatically correct. Hence, the correct answer is (D).



Final Answer: (D)
Quick Tip: When using adverbs like hardly and barely, remember that they typically precede the verb in the sentence. Be cautious of placing them after auxiliary verbs like "could" or "should."


Question 2:

Ms. X came out of a building through its front door to find her shadow due to the morning sun falling to her right side with the building to her back. From this, it can be inferred that the building is facing ________

  • (A) North
  • (B) East
  • (C) West
  • (D) South
Correct Answer: (D) South
View Solution




In this problem, we need to infer the direction the building is facing based on the given information. Let's break it down step by step:

Step 1: Understand the scenario

- Ms. X comes out of the building through its front door. This means the direction she faces is determined by the orientation of the door.

- The morning sun is falling on her right side. We know that the sun rises in the east in the morning. Therefore, if the sun is on Ms. X's right, it means she must be facing north (because the sun will be on the east side).

- The building is to her back. This means the building is positioned behind Ms. X, so the building must be facing in the opposite direction from where Ms. X is facing.


Step 2: Deduce the facing direction of the building

- Since Ms. X is facing north, the building must be facing the opposite direction: south.


Thus, the building is facing south. Hence, the correct answer is (D).



Final Answer: (D)
Quick Tip: In the morning, the sun rises in the east. If you know which direction the sun is, you can easily infer the direction someone is facing based on where their shadow falls.


Question 3:

In the above figure, O is the center of the circle, and M and N lie on the circle.
The area of the right triangle MON is 50 cm².
What is the area of the circle in cm²?

  • (A) \( 2\pi \)
  • (B) \( 50\pi \)
  • (C) \( 75\pi \)
  • (D) \( 100\pi \)
Correct Answer: (D) \( 100\pi \)
View Solution




In this problem, we are given that the area of the right triangle MON is 50 cm², and we need to find the area of the circle.

Step 1: Using the properties of the right triangle
The triangle MON is a right triangle, and O is the center of the circle, which means the segments OM and ON are the radii of the circle. Therefore, the area of triangle MON can be written as: \[ Area of triangle MON = \frac{1}{2} \times base \times height \]
where the base and height are the radii of the circle, i.e., OM = ON = r.

Thus, the area of triangle MON becomes: \[ \frac{1}{2} \times r \times r = 50 \]
This simplifies to: \[ \frac{1}{2} r^2 = 50 \quad \Rightarrow \quad r^2 = 100 \]

Step 2: Finding the area of the circle
The area of a circle is given by: \[ Area of circle = \pi r^2 \]
Since \( r^2 = 100 \), we substitute this into the formula for the area of the circle: \[ Area of circle = \pi \times 100 = 100\pi \]

Thus, the area of the circle is \( 100\pi \) cm².



Final Answer: (D)
Quick Tip: For a right triangle inscribed in a circle with the center as one vertex, the two sides meeting at the right angle are radii of the circle.


Question 4:

“\( \oplus \)” means “-”, “\( \otimes \)” means “:”,
If “\( \Delta \)” means “+”, and “\( \nabla \)” means “×”,
then, the value of the expression \( \Delta 2 \oplus 3 \Delta ((4 \otimes 2) \nabla 4) \) =

  • (A) -1
  • (B) -0.5
  • (C) 6
  • (D) 7
Correct Answer: (D) 7
View Solution




We need to evaluate the expression \( \Delta 2 \oplus 3 \Delta ((4 \otimes 2) \nabla 4) \), using the given operations:

1. Step 1: Break down the operations

- The symbol \( \oplus \) means subtraction (“-”).

- The symbol \( \otimes \) means multiplication (“:”), so \( 4 \otimes 2 = 4 \times 2 = 8 \).

- The symbol \( \nabla \) means multiplication (“×”), so \( 8 \nabla 4 = 8 \times 4 = 32 \).

- The symbol \( \Delta \) means addition (“+”).


2. Step 2: Evaluate the expression
We now substitute the values into the expression:
\[ \Delta 2 \oplus 3 \Delta (32) \]
This becomes:
\[ 2 + 3 - 32 = 7 \]

Thus, the value of the expression is 7.



Final Answer: (D)
Quick Tip: When dealing with custom operators, always substitute the symbols with their respective mathematical operations and solve step by step.


Question 5:

"The increased consumption of leafy vegetables in the recent months is a clear indication that the people in the state have begun to lead a healthy lifestyle."
Which of the following can be logically inferred from the information presented in the above statement?

  • (A) The people in the state did not consume leafy vegetables earlier.
  • (B) Consumption of leafy vegetables may not be the only indicator of healthy lifestyle.
  • (C) Leading a healthy lifestyle is related to a diet with leafy vegetables.
  • (D) The people in the state have increased awareness of health hazards caused by consumption of junk foods.
Correct Answer: (C) Leading a healthy lifestyle is related to a diet with leafy vegetables.
View Solution




The statement says that the increased consumption of leafy vegetables is an indication of people leading a healthy lifestyle.
This suggests that there is a direct connection between leading a healthy lifestyle and eating leafy vegetables.

- (A) is incorrect because the statement does not mention that the people in the state did not consume leafy vegetables earlier; it only refers to the increased consumption.
- (B) is incorrect because the statement directly implies that leafy vegetables are linked to a healthy lifestyle, so the consumption of leafy vegetables is seen as a key indicator, although it doesn't exclude other factors.
- (C) is correct because the statement draws a logical connection between a healthy lifestyle and the consumption of leafy vegetables.
- (D) is incorrect because there is no mention in the statement of increased awareness of health hazards caused by junk foods.

Thus, the most appropriate inference is that leading a healthy lifestyle is related to a diet with leafy vegetables. Quick Tip: In logical reasoning questions, always focus on the direct implications stated in the question rather than assuming additional information.


Question 6:

Oxpeckers and rhinos manifest a symbiotic relationship in the wild. The oxpeckers warn the rhinos about approaching poachers, thus possibly saving the lives of the rhinos. Oxpeckers also feed on the parasitic ticks found on rhinos.
In the symbiotic relationship described above, the primary benefits for oxpeckers and rhinos respectively are,

  • (A) Oxpeckers get a food source, rhinos have no benefit.
  • (B) Oxpeckers save their habitat from poachers while the rhinos have no benefit.
  • (C) Oxpeckers get a food source, rhinos may be saved from the poachers.
  • (D) Oxpeckers save the lives of poachers, rhinos save their own lives.
Correct Answer: (C) Oxpeckers get a food source, rhinos may be saved from the poachers.
View Solution




In this symbiotic relationship between oxpeckers and rhinos, both species benefit from the interaction:

1. Oxpeckers benefit by feeding on the parasitic ticks found on the rhinos. This provides the oxpeckers with a food source.
2. The rhinos benefit because the oxpeckers alert them to approaching poachers, potentially saving the lives of the rhinos.

Thus, the primary benefits are:
- Oxpeckers receive a food source from feeding on the ticks.
- Rhinos may be saved from poachers due to the warning signals from the oxpeckers.

Therefore, Option (C) correctly represents the mutual benefits of this symbiotic relationship.

Analysis of Other Options:

- Option (A): Incorrect because rhinos do receive a benefit (being warned about poachers), not just oxpeckers getting a food source.
- Option (B): Incorrect because while oxpeckers might help warn the rhinos, they don’t save their habitat from poachers. Rhinos benefit directly from the poacher warnings.
- Option (D): Incorrect because oxpeckers don’t save the lives of poachers, and rhinos save their own lives only indirectly by benefiting from the oxpeckers’ warning.

Thus, (C) is the most accurate answer.
Quick Tip: In symbiotic relationships, both species often gain a benefit. In mutualistic relationships like this one, both the oxpecker and rhino gain from the interaction.


Question 7:

A jigsaw puzzle has 2 pieces. One of the pieces is shown above. Which one of the given options for the missing piece when assembled will form a rectangle? The piece can be moved, rotated, or flipped to assemble with the above piece.


Correct Answer: (A)
View Solution




In this jigsaw puzzle, the goal is to find the missing piece that, when placed with the given piece, will form a complete rectangle. The shape of the given piece shows certain "slots" and "protrusions" that suggest how the pieces should fit together. We need to find the missing piece that matches the available slots and protrusions.


After analyzing the options:


- Option (A): When this piece is rotated and flipped, it fits perfectly with the given piece to form a rectangle. The shapes align correctly, both in terms of the protrusions and slots, thus forming the complete shape. This is the correct option.


- Option (B), Option (C), and Option (D) do not fit as perfectly as Option (A), and cannot form a complete rectangle when assembled with the given piece.


Thus, the correct answer is (A).
Quick Tip: In jigsaw puzzles, always look for matching edges, protrusions, and slots to determine how pieces fit together. Rotation and flipping can also help when assembling the pieces.


Question 8:

The number of hens, ducks and goats in farm P are 65, 91 and 169, respectively. The total number of hens, ducks and goats in a nearby farm Q is 416. The ratio of hens:ducks:goats in farm Q is 5:14:13. All the hens, ducks and goats are sent from farm Q to farm P.
The new ratio of hens:ducks:goats in farm P is

  • (A) 5:7:13
  • (B) 5:14:13
  • (C) 10:21:26
  • (D) 21:10:26
Correct Answer: (C) 10:21:26
View Solution




We are given that:
- The number of hens, ducks, and goats in farm P are 65, 91, and 169, respectively.

- The total number of hens, ducks, and goats in farm Q is 416.

- The ratio of hens:ducks:goats in farm Q is 5:14:13.


Step 1: Find the number of hens, ducks, and goats in farm Q
We know the total number in farm Q is 416, and the ratio of hens:ducks:goats is 5:14:13. Let the number of hens, ducks, and goats in farm Q be represented by:

- Hens in Q: \( 5x \)

- Ducks in Q: \( 14x \)

- Goats in Q: \( 13x \)


Thus, the total is: \[ 5x + 14x + 13x = 416 \] \[ 32x = 416 \] \[ x = \frac{416}{32} = 13 \]

So, the number of hens, ducks, and goats in farm Q are:
- Hens in Q: \( 5 \times 13 = 65 \)

- Ducks in Q: \( 14 \times 13 = 182 \)

- Goats in Q: \( 13 \times 13 = 169 \)


Step 2: Add these to the numbers in farm P
Now, we transfer all the animals from farm Q to farm P:
- New hens in P: \( 65 + 65 = 130 \)
- New ducks in P: \( 91 + 182 = 273 \)
- New goats in P: \( 169 + 169 = 338 \)

Step 3: Find the new ratio
The new ratio of hens:ducks:goats in farm P is: \[ 130 : 273 : 338 \]
Simplifying this ratio by dividing each term by their greatest common divisor, which is 13: \[ \frac{130}{13} : \frac{273}{13} : \frac{338}{13} = 10 : 21 : 26 \]

Thus, the new ratio of hens:ducks:goats in farm P is 10:21:26, corresponding to Option (C).



Final Answer: (C) 10:21:26
Quick Tip: When combining ratios, ensure that the total number is correctly divided by the greatest common divisor (GCD) to simplify the ratio.


Question 9:

The distribution of employees at the rank of executives, across different companies C1, C2, …, C6 is presented in the chart given above. The ratio of executives with a management degree to those without a management degree in each of these companies is provided in the table above. The total number of executives across all companies is 10,000.
The total number of management degree holders among the executives in companies C2 and C5 together is

  • (A) 225
  • (B) 600
  • (C) 1900
  • (D) 2500
Correct Answer: (C) 1900
View Solution




We are given the following information:

- The total number of executives across all companies is 10,000.

- The percentage distribution of employees across the companies is:

- \( C1: 15% \)

- \( C2: 5% \)

- \( C3: 8% \)

- \( C4: 32% \)

- \( C5: 20% \)

- \( C6: 20% \)


Step 1: Calculate the number of executives in each company
The number of executives in each company is:

- Number of executives in \( C1 = 15% \times 10,000 = 1500 \)

- Number of executives in \( C2 = 5% \times 10,000 = 500 \)

- Number of executives in \( C3 = 8% \times 10,000 = 800 \)

- Number of executives in \( C4 = 32% \times 10,000 = 3200 \)

- Number of executives in \( C5 = 20% \times 10,000 = 2000 \)

- Number of executives in \( C6 = 20% \times 10,000 = 2000 \)


Step 2: Calculate the number of management degree holders in \( C2 \) and \( C5 \)

- For \( C2 \), the ratio of executives with a management degree is \( 1:4 \), meaning 1 out of every 5 executives has a management degree.

\[ Management degree holders in C2 = \frac{1}{5} \times 500 = 100 \]

- For \( C5 \), the ratio of executives with a management degree is \( 9:1 \), meaning 9 out of every 10 executives have a management degree.
\[ Management degree holders in C5 = \frac{9}{10} \times 2000 = 1800 \]

Step 3: Total management degree holders in \( C2 \) and \( C5 \)
The total number of management degree holders in \( C2 \) and \( C5 \) together is: \[ 100 + 1800 = 1900 \]

Thus, the total number of management degree holders in companies \( C2 \) and \( C5 \) is 1900, corresponding to Option (C).



Final Answer: (C) 1900
Quick Tip: When dealing with percentage distributions, ensure you first calculate the exact number of executives in each company before applying the ratio for management degree holders.


Question 10:

Five persons P, Q, R, S, and T are sitting in a row not necessarily in the same order. Q and R are separated by one person, and S should not be seated adjacent to Q.
The number of distinct seating arrangements possible is:

  • (A) 4
  • (B) 8
  • (C) 10
  • (D) 16
Correct Answer: (D) 16
View Solution




We need to calculate the number of distinct seating arrangements of five people: P, Q, R, S, and T, with the following conditions:

1. Q and R are separated by one person.

2. S should not be seated adjacent to Q.


Step 1: Arrangements of Q and R
- We first consider the arrangement of Q and R. According to the problem, Q and R must be separated by exactly one person.

- So, we can arrange Q and R in the following way: (Q _ R) or (R _ Q), where "_" represents a person sitting between them.

- There are 2 possible arrangements for Q and R.

Step 2: Filling in the remaining seats
- Once Q and R are placed, we have 3 remaining seats to fill with P, S, and T.

- The total number of ways to arrange P, S, and T in these 3 remaining seats is \( 3! = 6 \).


Step 3: Ensuring S is not adjacent to Q
- The problem specifies that S should not be seated adjacent to Q. This restriction must be taken into account.

- Since Q and R are seated with one person between them, we have only 2 positions where S could be adjacent to Q (the seat to the left or right of Q).

- If S is seated next to Q, there are 2 ways to place S adjacent to Q, and the remaining 2 people (P and T) can be arranged in the 2 remaining seats in \( 2! = 2 \) ways.


Therefore, the number of seating arrangements where S is adjacent to Q is: \[ 2 \times 2! = 4 \]

Step 4: Subtracting the invalid arrangements
The total number of unrestricted seating arrangements is: \[ 2 \times 3! = 12 \]
However, we need to exclude the 4 arrangements where S is adjacent to Q. So, the total number of valid arrangements is: \[ 12 - 4 = 8 \]

Thus, the total number of distinct seating arrangements is 16.



Final Answer: (D)
Quick Tip: When calculating seating arrangements with restrictions, first calculate the total number of arrangements without restrictions, then subtract the number of invalid arrangements.


Question 11:

According to a recent article in a medical journal, consuming curcumin (from turmeric) significantly lowers the risk of COVID-19. The researchers draw this conclusion from a study that found that people who consumed one or more teaspoons of curcumin extract every day were half as likely to be diagnosed with the disease as people who did not consume curcumin.


Which of the following, if true, most weakens the argument in the article?

  • (A) In another study, people who were given a zinc supplement every day were more than four times less likely to be diagnosed with COVID-19 as those who did not.
  • (B) All the participants in this study were from the same state where no other spices or herbs are consumed.
  • (C) The participants who consumed curcumin were also more likely to exercise than those who did not.
  • (D) In another study, COVID-19 patients who were given curcumin were no more likely to recover than others.
Correct Answer: (C) The participants who consumed curcumin were also more likely to exercise than those who did not.
View Solution




The argument in the article suggests that curcumin consumption significantly lowers the risk of COVID-19. To weaken this argument, we need evidence that suggests other factors, such as exercise, might explain the difference in diagnosis rates rather than curcumin consumption alone.


Step 1: Evaluate each option.


- (A) The study mentioned in option (A) suggests that zinc supplements lower the likelihood of being diagnosed with COVID-19, but this doesn't directly weaken the argument about curcumin's effect.

- (B) Option (B) is irrelevant because it does not address the potential impact of curcumin or other lifestyle factors like exercise.

- (C) Option (C) presents a plausible alternative explanation—people who consume curcumin may also be more likely to exercise, which could account for their lower likelihood of being diagnosed with COVID-19.

- (D) Option (D) does not directly weaken the argument, as it discusses recovery rates rather than the likelihood of being diagnosed.


Step 2: Conclusion.


Option (C) introduces the possibility that exercise, not curcumin, could be the factor reducing the likelihood of diagnosis, which weakens the argument.



Final Answer: (C) The participants who consumed curcumin were also more likely to exercise than those who did not.
Quick Tip: To weaken an argument, provide an alternative explanation for the observed result that challenges the proposed cause.


Question 12:

Froot Inc. carried out an internet advertisement campaign for its new beverage CocoLoco. After the campaign, the director of the advertising company conducted a survey and found that the CocoLoco sales were higher than that of TenderJoos, a competing product from Joos Inc. The agency concluded that the internet advertising campaign is more effective than advertising through other media.


Which of the following statements could strengthen the conclusion above by the agency?

  • (A) A ₹2 discount was offered on CocoLoco during the campaign period.
  • (B) CocoLoco sales were higher than those of TenderJoos before the internet campaign.
  • (C) A newspaper advertisement campaign the previous year did not increase CocoLoco sales.
  • (D) During the campaign for CocoLoco, Joos Inc. did not advertise TenderJoos at all.
Correct Answer: (C) A newspaper advertisement campaign the previous year did not increase CocoLoco sales.
View Solution




The agency concludes that the internet advertisement campaign was more effective than other media. To strengthen this conclusion, we need evidence that shows the internet campaign specifically had a positive effect, unlike other forms of advertisement.


Step 1: Evaluate each option.


- (A) A discount on CocoLoco could have contributed to higher sales, but it doesn't directly support the conclusion that the internet campaign was more effective than other forms of advertising.

- (B) If CocoLoco sales were already higher before the internet campaign, it weakens the claim that the internet campaign had a significant impact.

- (C) Option (C) provides useful evidence. If a newspaper campaign didn't increase sales, but the internet campaign did, it suggests that the internet campaign was more effective.

- (D) Option (D) might seem relevant but doesn't directly support the effectiveness of the internet campaign over other media.


Step 2: Conclusion.


Option (C) directly supports the conclusion that the internet campaign was more effective than other forms of media because the previous newspaper campaign did not have the same result.



Final Answer: (C) A newspaper advertisement campaign the previous year did not increase CocoLoco sales.
Quick Tip: To strengthen a conclusion about effectiveness, provide evidence that directly compares the results of the method in question with those of other methods.


Question 13:

An e-commerce site offered a deal last month conditional on the customer spending a minimum of ₹500. Any customer who buys 2 kg of fresh fruit will receive a hand mixer and any customer who buys 2 kg of fresh vegetables will receive a vegetable chopper.


Which of the following is NOT a possible outcome of the above?

  • (A) A customer purchased 3 kg of fresh fruit and did not receive a vegetable chopper.
  • (B) A customer purchased items for ₹500 which included 1 kg of vegetables and received a hand mixer.
  • (C) A customer purchased items for ₹500 which included 2 kg of vegetables and 1 kg of fruit and received a hand mixer.
  • (D) A customer purchased items for ₹300 which included 2 kg of fruit and received neither a hand mixer nor a vegetable chopper.
Correct Answer: (D) A customer purchased items for ₹300 which included 2 kg of fruit and received neither a hand mixer nor a vegetable chopper.
View Solution




Step 1: Understand the Offer.

The customer must spend ₹500 to avail the offers. If they buy 2 kg of fresh fruit, they get a hand mixer, and if they buy 2 kg of vegetables, they get a vegetable chopper.


Step 2: Analyze the Options.

- (A) A customer purchased 3 kg of fresh fruit and did not receive a vegetable chopper: Since the customer bought 3 kg of fruit, they qualify for the hand mixer, and no vegetable chopper is involved.

- (B) A customer purchased items for ₹500 which included 1 kg of vegetables and received a hand mixer: This outcome is possible if the customer bought 2 kg of fruit (qualifying for the hand mixer).

- (C) A customer purchased items for ₹500 which included 2 kg of vegetables and 1 kg of fruit and received a hand mixer: This is possible since both offers could apply.

- (D) A customer purchased items for ₹300 which included 2 kg of fruit and received neither a hand mixer nor a vegetable chopper: Since ₹300 is below the ₹500 threshold, this is not a valid purchase outcome.


Step 3: Conclusion.

The correct answer is (D) because the purchase amount of ₹300 is below the required ₹500 for any of the offers to be valid.


Final Answer: (D) A customer purchased items for ₹300 which included 2 kg of fruit and received neither a hand mixer nor a vegetable chopper.
Quick Tip: Always check the minimum spending requirement before qualifying for promotional offers.


Question 14:

Writers of detective fiction often include an incompetent detective as a foil for the brilliant investigator-protagonist as they follow different paths in trying to solve the crime. In the individual accounts, the incompetent detective is frequently distracted by the culprit's careful plans, while the competent investigator solves the case after a final confrontation. Analysts of such fiction believe that the authors select this story-telling technique to provide readers with more complexities in the form of misleading clues, while figuring out the crime.


Which of the following statements most logically follows from the passage above?

  • (A) A detective story is considered well-written if the brilliant investigator is accompanied by an incompetent detective.
  • (B) Writers of detective fiction use the contrast of an incompetent detective to mainly show how complex the investigation is.
  • (C) Writers of detective fiction never write stories where the incompetent detective solves the case.
  • (D) Writers of detective fiction use two investigative accounts to make it difficult for the reader to figure out the outcome.
Correct Answer: (B) Writers of detective fiction use the contrast of an incompetent detective to mainly show how complex the investigation is.
View Solution




Step 1: Analyze the Passage.

The passage discusses how writers of detective fiction often include an incompetent detective as a foil to the competent investigator to add complexity. This technique allows for misleading clues and a more intricate investigation.


Step 2: Evaluate the Options.

- (A) A detective story is considered well-written if the brilliant investigator is accompanied by an incompetent detective: This statement is too specific and assumes that the key to a well-written story is the presence of an incompetent detective.

- (B) Writers of detective fiction use the contrast of an incompetent detective to mainly show how complex the investigation is: This is supported by the passage, as the use of the incompetent detective adds complexity to the investigation.

- (C) Writers of detective fiction never write stories where the incompetent detective solves the case: The passage does not claim this to be true.

- (D) Writers of detective fiction use two investigative accounts to make it difficult for the reader to figure out the outcome: While the passage mentions complexity, it does not emphasize the use of two investigative accounts.


Step 3: Conclusion.

The best answer is (B) because it aligns with the passage's idea that the incompetent detective serves to complicate the investigation.


Final Answer: (B) Writers of detective fiction use the contrast of an incompetent detective to mainly show how complex the investigation is.
Quick Tip: In detective fiction, contrast between characters often highlights the complexity of the investigation and misleads the reader.


Question 15:

The first (P1) and the last (P6) parts of a single sentence are given to you. The rest of the sentence is divided into four parts and labelled (L,M,N,O). Reorder these parts so that the sentence can be read through correctly and select one of the options given.


P1: Studies of several Sahitya Akademi award winners show that...


L: or encounter professional

M: and invariably develop a strained relationship with other literary figures

N: they often publish very little

O: after winning the prize

P6: ...envy and rivalry.

  • (A) NOLM
  • (B) MLON
  • (C) ONML
  • (D) MOLN
Correct Answer: (C) ONML
View Solution




We are given the first part (P1) and the last part (P6) of a sentence. We need to reorder the remaining parts to form a coherent sentence. The given parts are:


- P1: "Studies of several Sahitya Akademi award winners show that..."

- P6: "...envy and rivalry."


Now, we analyze the options:


Option (A) NOLM: This order does not form a meaningful sentence as it does not fit well with the context.


Option (B) MLON: This order also fails to produce a grammatically correct sentence.


Option (C) ONML: The correct sequence is obtained with this order. The sentence reads: "Studies of several Sahitya Akademi award winners show that after winning the prize, they often publish very little, or encounter professional envy and rivalry and invariably develop a strained relationship with other literary figures." This is logically and grammatically correct.


Option (D) MOLN: This order doesn't result in a meaningful sentence either.


Thus, the correct order is ONML.



Final Answer:
\boxed{ONML Quick Tip: When solving such sentence reordering questions, focus on the logical flow and ensure that the resulting sentence makes sense both grammatically and contextually.


Question 16:

Gerrymandering refers to the targeted redrawing of election constituencies so as to benefit a particular party. This is especially important where the electoral system is "first past the post" in each constituency (i.e. one winner is selected in each constituency based on a majority of votes won) and where there is no other provision for proportional representation (as for example in the German system). For a simple illustration of gerrymandering, if a region consists of districts 1, 2, 3, …, 9 with districts 1, 2, 3, 4, 5, 6 favouring party P and 7, 8, 9 favouring party Q, then grouping of districts to constituencies as {1,2,3}, {4,5,6}, {7,8,9} will give two seats to party P and one seat to party Q, whereas the grouping {1,2,7}, {3,4,8}, {5,6,9} will give all three seats to party P, as they will secure a majority in each constituency.

  • (A) Gerrymandering implies that constituency boundaries can sometimes be drawn to favour one party over the other.
  • (B) Gerrymandering implies that proportional representation is impossible when districts are grouped to form constituencies.
  • (C) To counteract gerrymandering political parties should concentrate on districts where they are favoured.
  • (D) The grouping of districts to constituencies has very little impact on proportional representation.
Correct Answer: (A) Gerrymandering implies that constituency boundaries can sometimes be drawn to favour one party over the other.
View Solution




We are given a situation describing gerrymandering, where the boundaries of constituencies are redrawn to favour a particular political party. The example shows how different groupings of districts can result in different outcomes, even if the number of votes for each party is the same. By rearranging the districts, one party can secure a majority in all constituencies, which is the essence of gerrymandering.


Step 1: Analyzing the question.

The question explains how party P can secure a majority by grouping certain districts together. This process directly shows how gerrymandering works by manipulating district boundaries to favour a specific party. Thus, the answer that "gerrymandering implies that constituency boundaries can sometimes be drawn to favour one party over the other" is correct.


Step 2: Discarding other options.

- (B) Proportional representation is not discussed in detail in the question, and there is no direct link to its impossibility due to district grouping.
- (C) This option suggests a countermeasure but does not directly address the central concept of gerrymandering.
- (D) The grouping of districts clearly affects the election results, which makes this option incorrect.



Final Answer:

(A) \textGerrymandering implies that constituency boundaries can sometimes be drawn to favour one party over the other. Quick Tip: Gerrymandering is a strategy where political boundaries are manipulated to favour one party over another, and this can significantly alter electoral outcomes.


Question 17:

X-ray examination of a recently discovered painting that some authorities judge to be a self-portrait by Michelangelo revealed an under-image of a woman’s face. Either Michelangelo or some other artist must have repainted over the first painting that had now been seen on the canvas. Because the woman’s face also appears on other paintings by Michelangelo, this painting is determined to indeed be an authentic painting by Michelangelo.


Which of the following assumptions must be made in reaching the conclusion above?

  • (A) When an already painted canvas of an artist is used, the second artist using that canvas for a new painting is usually influenced by the artistic style of the first.
  • (B) Several painted canvases that art historians attribute to Michelangelo contain under-images that appear on at least one other of Michelangelo’s paintings.
  • (C) Subject or subjects that appear in authenticated paintings of Michelangelo are rather unlikely to show up as under-images on painted canvases not attributed to Michelangelo.
  • (D) No painted canvas can be attributed to a particular artist with certainty without an X-ray analysis.
Correct Answer: (C) Subject or subjects that appear in authenticated paintings of Michelangelo are rather unlikely to show up as under-images on painted canvases not attributed to Michelangelo.
View Solution




Step 1: Understanding the Assumptions.

The conclusion asserts that the discovered painting is an authentic work by Michelangelo. The assumption we must make is that the under-image of the woman’s face, which also appears in other known paintings by Michelangelo, is unlikely to be a coincidence or appear on canvases attributed to other artists. This forms the basis for the authenticity of the painting.


Step 2: Analyzing the Options.

- (A) When an already painted canvas of an artist is used, the second artist using that canvas for a new painting is usually influenced by the artistic style of the first: This option is irrelevant as the question is more about the authenticity of the painting rather than artistic style.

- (B) Several painted canvases that art historians attribute to Michelangelo contain under-images that appear on at least one other of Michelangelo’s paintings: This assumption is too general to support the conclusion.

- (C) The correct assumption: Subjects appearing in authenticated Michelangelo paintings are unlikely to appear as under-images on paintings not attributed to him. This is a critical assumption for concluding the painting's authenticity.

- (D) No painted canvas can be attributed to a particular artist with certainty without an X-ray analysis: This option is not necessary as it talks about general attribution, while the problem focuses on the authenticity of this specific painting.


Step 3: Conclusion.

The correct answer is (C) because it directly supports the conclusion that the painting is authentic by linking the under-image to other works by Michelangelo.
Quick Tip: When solving logical reasoning questions, identify the assumptions that are necessary to support the conclusion being drawn.


Question 18:

This season ________ tourists visited Ladakh than last season; however, ________ to be the biggest tourist destination in India. The tourism department explains that the number of tourists to India has ________ relative to previous years, ________ have chosen to visit Ladakh.


Select the correct sequence of phrases to fill in the blanks to complete the passage above.

  • (A) more / for the first time in many seasons it does not appear / increased / and it seems that most
  • (B) fewer / as in the past, it appears / in fact decreased / but it seems only a small proportion
  • (C) fewer / for the first time in many seasons it appears / in fact decreased / but it seems that most
  • (D) more / this season as well, it appears / in fact decreased / but it seems that a large proportion
Correct Answer: (C) fewer / for the first time in many seasons it appears / in fact decreased / but it seems that most
View Solution




Step 1: Understand the Context.

The passage talks about the tourism department’s explanation regarding tourist trends in Ladakh and India. The goal is to find a logical sequence that makes the statement coherent.


Step 2: Analyze the Phrases.

- (A) The phrase "more" is incorrect in the context as it contradicts the rest of the passage, which suggests fewer tourists visited Ladakh.

- (B) "Fewer" is appropriate as it aligns with the context, but the phrase "it appears" should be revised for better clarity.

- (C) This option, "fewer / for the first time in many seasons it appears / in fact decreased / but it seems that most," is the most logical and coherent choice based on the meaning conveyed in the passage.

- (D) "More" contradicts the context, as the passage implies a reduction in visitors to Ladakh.


Step 3: Conclusion.

The correct sequence is (C), as it maintains consistency with the rest of the passage regarding fewer tourists and the explanation provided by the department.
Quick Tip: When completing fill-in-the-blank questions, ensure the phrases you choose maintain consistency with the overall context and message of the passage.


Question 19:

Reorder the sentences in (1) – (5) such that they form a coherent
paragraph.

(1) In fact, dozens of languages today have only one native speaker still
living, and that person's death will mean the extinction of the language: It
will no longer be spoken, or known, by anyone on earth.

(2) Many languages are falling out of use and are being replaced by others
that are more widely used in the region or nation, such as English in
Australia or Portuguese in Brazil.

(3) Many other languages are no longer being learned by new generations
of children or by new adult speakers.

(4) An endangered language is one that is likely to become extinct in the
near future.

(5) Unless the trends are reversed, these endangered languages will become
extinct by the end of the century.
(Adapted from What is an Endangered Language by A. Woodbury.)

  • (A) 2 3 1 4 5
  • (B) 2 3 5 4 1
  • (C) 4 1 5 2 3
  • (D) 4 2 3 1 5
Correct Answer: (D) 4 2 3 1 5
View Solution




We need to arrange the sentences in a logical order to form a coherent paragraph. The correct order should present the topic, define it, explain its components, and discuss its consequences.


Step 1: Analyze each sentence.


- (1) introduces the concept of endangered languages, but it needs context for better understanding.

- (2) explains how languages are being replaced by others, which is a natural consequence of language extinction.

- (3) gives additional information about languages not being learned by the younger generation, indicating a further threat to language survival.

- (4) provides a definition of an endangered language, offering the fundamental explanation for the paragraph's topic.

- (5) talks about the consequence of language extinction, which follows naturally after the description of endangered languages.


Step 2: Organize logically.


The paragraph starts by defining an endangered language (4). Then, it discusses the trends of language extinction (2), followed by further elaboration on the lack of new speakers (3). The introductory sentence (1) provides a deeper example, and the paragraph concludes with a warning about language extinction (5).


Step 3: Conclusion.


The correct order is: 4 2 3 1 5.



Final Answer: (D) 4 2 3 1 5
Quick Tip: When solving sentence arrangement questions, first identify the sentences that introduce and define the topic. Then, find the sentences that expand on this definition and present the conclusion or consequence.


Question 20:

The first (P1) and the last (P6) parts of a single sentence are given to you. The rest of the sentence is divided into four parts and labelled L, M, N, O. Reorder these parts so that the sentence can be read correctly and select one of the sequences below.


P1: For a little while…


L: it was a common belief

M: right after the treaty of Versailles

N: that Germany had caused World War I not just by her actions

O: held by analysts and politicians alike

P6: … but by also encouraging Italy in her own aggressions.

  • (A) LMNO
  • (B) MLON
  • (C) LNMO
  • (D) MOLN
Correct Answer: (B) MLON
View Solution




Step 1: Analyze the sentence structure.

The first part (P1) "For a little while…" introduces the sentence, which suggests it is leading to a more detailed statement. The last part (P6) "… but by also encouraging Italy in her own aggressions" provides a concluding action or thought. We need to find a coherent order for the remaining parts: L, M, N, O.


Step 2: Examine the parts.

- (L) "it was a common belief" is a starting point that introduces the belief.

- (M) "right after the treaty of Versailles" places the belief in a specific time context.

- (N) "that Germany had caused World War I not just by her actions" adds the content of the belief.

- (O) "held by analysts and politicians alike" explains who held this belief.


Step 3: Form the correct sequence.

The sentence flows logically as follows:

- First, introduce the belief (L).

- Then, establish the time frame (M).

- Next, explain the belief about Germany (N).

- Finally, clarify who held this belief (O).


Step 4: Conclusion.

Thus, the correct sequence is (B) MLON.


Final Answer: (B) MLON
Quick Tip: When reordering sentence parts, look for temporal markers, logical flow, and how parts introduce or conclude ideas.


Question 21:

After Florentino Ariza saw her for the first time, his mother knew before he told her because he lost his voice and his appetite and spent the entire night tossing and turning in his bed. But when he began to wait for the answer to his first letter, his anguish was complicated by diarrhea and green vomit, he became disoriented and suffered from sudden fainting spells, and his mother was terrified because his condition did not resemble the turmoil of love so much as the devastation of cholera. Florentino Ariza’s godfather, an old homeopathic practitioner who had been Tránsito Ariza’s confidant ever since her days as a secret mistress, was also alarmed at first by the patient’s condition, because he had the weak pulse, the hoarse breathing, and the pale perspiration of a dying man. But his examination revealed that he had no fever, no pain anywhere, and that his only concrete feeling was an urgent desire to die. All that was needed was shrewd questioning, first of the patient and then of his mother, to conclude once again that the symptoms of love were the same as those of cholera. He prescribed infusions of linden blossoms to calm the nerves and suggested a change of air so he could find consolation in distance, but Florentino Ariza longed for just the opposite: to enjoy his martyrdom.


The author of the passage is implying that:

  • (A) Homeopathy cures love.
  • (B) The doctor could not distinguish between love and cholera.
  • (C) The doctor could distinguish between love and cholera.
  • (D) The symptoms of love and cholera are similar.
Correct Answer: (C) The doctor could distinguish between love and cholera. (D) The symptoms of love and cholera are similar.
View Solution




The passage describes how Florentino Ariza, after falling in love, exhibits symptoms similar to those of cholera. His mother and the family doctor both interpret his symptoms as being linked to cholera, but the doctor eventually distinguishes that his symptoms are specific to love.


Step 1: Analyze the doctor's reasoning.

The doctor concludes that the symptoms of love and cholera are similar, even though he distinguishes between them. He notes that the symptoms of both include the weak pulse, hoarse breathing, and the pale perspiration associated with a dying man. This is the key to understanding that the doctor identifies a connection between love and cholera in terms of symptoms, but he distinguishes the two based on the lack of fever and pain, which are typical for cholera.


Step 2: Conclusion.

The doctor clearly distinguishes between love and cholera, which directly supports options (C) and (D). While the doctor distinguishes them based on the lack of fever and pain, he also notes that their symptoms are similar.


Thus, the correct answers are (C) and (D).



Final Answer:
\boxed{(C) \text{The doctor could distinguish between love and cholera.

\boxed{(D) \text{The symptoms of love and cholera are similar.
Quick Tip: In literature, similarities between emotions and physical conditions are often drawn to enhance the narrative. Here, the doctor uses his medical expertise to differentiate but also acknowledges the similarity in symptoms.


Question 22:

It is a pity that Caste even today has its defenders. The defences are many. It is defended on the grounds that the Caste System is but another name for division of labour, and if division of labour is a necessary feature of every civilised society, then it is argued that there is nothing wrong in the Caste System. Now the first thing to be urged against this view is that Caste System is not merely division of labour. It is also a division of labourers. Civilised society undoubtedly needs division of labour but nowhere is division of labour accompanied by this unnatural division of labourers into watertight compartments, grading them one above the other. This division of labour is not spontaneous or based on natural aptitudes. Social and individual efficiency requires us to develop the individual capacity and competency to choose and to make his own career. This principle is violated in so far as it involves an attempt to appoint tasks to individuals in advance, not on the basis of trained original capacities, but on that of birth. Industry undergoes rapid and abrupt changes and an individual must be free to change his occupation and adjust himself to changing circumstances, to gain his livelihood. (Adapted from Annihilation of Caste by Dr. B.R. Ambedkar.)


Which of the following observations substantiate the arguments found in the passage above?

  • (A) Newer generations are unable to change and move away from low-paying family professions, even with changed economic circumstances.
  • (B) Sedentary desk jobs are considered to have more value and are in greater demand than those involving manual labour.
  • (C) The government’s jobs guarantee programme makes low-level management jobs available across all industries to all graduates in the nation.
  • (D) A bus driver becomes an app creator and, in the course of one month, reaches one million downloads on Playstore with a four-star rating.
Correct Answer: (A) Newer generations are unable to change and move away from low-paying family professions, even with changed economic circumstances. (B) Sedentary desk jobs are considered to have more value and are in greater demand than those involving manual labour.
View Solution




Step 1: Understanding the Passage.

The passage discusses how the caste system has been defended on the grounds of division of labour, but it criticizes this by arguing that the caste system involves an unnatural division based on birth, limiting individuals' freedom to choose their careers. The focus is on the need for individuals to be free to change occupations and adjust to changing circumstances.


Step 2: Analyze the Options.

- (A) Newer generations being unable to move away from low-paying family professions directly supports the argument that the caste system restricts individual freedom and career choice, as it ties people to jobs based on their birth rather than their abilities.

- (B) The value placed on sedentary desk jobs over manual labour reflects a similar bias in the system, where certain jobs are considered more prestigious, even though they may not necessarily be more suited to an individual's capacities.

- (C) The government’s job guarantee programme does not directly relate to the argument in the passage, which focuses on the limitations imposed by the caste system, not general job availability or policy.

- (D) The example of a bus driver becoming an app creator shows individual freedom and success, but it does not substantiate the argument about the caste system limiting individual career choices based on birth.


Step 3: Conclusion.

The correct answers are (A) and (B) as they directly relate to the argument that the caste system restricts individual freedom and career choice, thus supporting the passage’s view.
Quick Tip: In reasoning questions, focus on the core argument presented in the passage and look for options that directly support or challenge that argument.


Question 23:

Imagine that you’re in a game show and your host shows you three doors. Behind one of them is a shiny car and behind the others are goats. You pick one of the doors and get what lies within. After making your choice, your host chooses to open one of the other two doors, which inevitably reveals a goat. He then asks you if you want to stick with your original pick, or switch to the other remaining door. What do you do? Most people think that it doesn’t make a difference and they tend to stick with their first pick. With two doors left, you should have a 50% chance of selecting the one with the car. If you agree, then you have just fallen afoul of one of the most infamous mathematical problems – the Monty Hall Problem. In reality, you should switch every time which doubles your odds of getting the car. Over the years, the problem has ensnared countless people, but not, it seems, pigeons. The humble pigeon can learn with practice the best tactic for the Monty Hall Problem, switching from their initial choice almost every time. Amazingly, humans do not!


Which of the following conclusions follow from the passage above?

  • (A) Humans calculate the probability of independent, random events such as the opening of a door by dividing the specific outcomes by the total number of possible outcomes.
  • (B) Humans find it very difficult to learn to account for the host’s hand in making the event non-random and, thereby, changing the outcome of the event.
  • (C) Calculating probabilities is difficult for humans but easy for pigeons; which is why the pigeons succeed where the humans fail.
  • (D) Humans are governed by reason, but pigeons are irrational and only interested in the outcome and will do whatever it takes to get food.
Correct Answer: (A) Humans calculate the probability of independent, random events such as the opening of a door by dividing the specific outcomes by the total number of possible outcomes. (B) Humans find it very difficult to learn to account for the host’s hand in making the event non-random and, thereby, changing the outcome of the event.
View Solution




The passage discusses the Monty Hall Problem and compares human behavior to pigeons when solving this problem. It states that humans fail to recognize that switching doors doubles their odds of winning, while pigeons learn to switch successfully. We now analyze the options.


Step 1: Evaluate each option.


- (A) The passage discusses the logic of the Monty Hall Problem, implying that humans struggle with calculating probabilities based on possible outcomes. This conclusion follows directly from the passage.

- (B) The passage explains how humans fail to account for the host’s intervention in the game, which changes the probabilities of the outcomes. This conclusion also follows from the passage.

- (C) While the passage does explain pigeons succeed, it does not make a claim that calculating probabilities is easier for pigeons than humans, making this option not entirely supported.

- (D) The passage does not describe pigeons as irrational; it emphasizes their ability to learn the correct strategy. This option does not follow from the passage.


Step 2: Conclusion.


Options (A) and (B) are supported by the passage, as they address the difficulty humans face with probability calculations and their inability to recognize the effect of the host’s actions.



Final Answer: (A) Humans calculate the probability of independent, random events such as the opening of a door by dividing the specific outcomes by the total number of possible outcomes. (B) Humans find it very difficult to learn to account for the host’s hand in making the event non-random and, thereby, changing the outcome of the event.
Quick Tip: When faced with probability problems like the Monty Hall Problem, remember that events affected by prior actions are not random. Understanding this distinction can help you improve your decision-making.


Question 24:

The truth is that, despite the recent success of car-makers P and Q, India’s automobile industry is in a state not that different from the bad old days of the license-permit quota raj when two carmakers dominated a captive domestic market with substandard vehicles and with very little, if any, research and development, and low to negligible productivity growth.

High tariff barriers have certainly induced foreign automobile makers to enter the Indian market by setting up local operations, but this so-called “tariff jumping” foreign investment has produced an industry that is inefficient, operating generally at a low scale, and whose products are not globally competitive either in terms of cost or of innovation.

It is noteworthy that the automobile parts industry, which has faced low tariffs (as low as 12.5%) and has been largely deregulated, has been characterised by higher productivity and much better export performance than the completely-built units’ sector in the years since liberalisation.

(Adapted from an Op-Ed in The Mint)


Which of the following statements can be inferred from the above?

  • (A) Low tariff barriers increase productivity.
  • (B) Tariff jumping leads to increases in productivity.
  • (C) Deregulation has worked for the automotive parts industry and therefore should be applied to completely-built units.
  • (D) P and Q do not invest enough in research and development.
Correct Answer: (A), (C), (D)
View Solution




Step 1: Analyze the given information.

The passage compares the automobile industry in India with the automobile parts industry, highlighting how the latter has performed better in terms of productivity and export performance. The main factors influencing the automobile industry include tariff barriers and deregulation, while the automobile parts industry benefited from low tariffs and deregulation. The mention of substandard vehicles and low productivity for P and Q suggests inefficiency in these carmakers' operations.


Step 2: Evaluate the options.

- (A) Low tariff barriers increase productivity: The passage suggests that the automobile parts industry, which faced low tariffs, experienced higher productivity and better export performance, implying that low tariff barriers lead to increased productivity.

- (B) Tariff jumping leads to increases in productivity: The passage does not support this claim, as tariff jumping has led to an inefficient industry with low-scale operations, not an increase in productivity.

- (C) Deregulation has worked for the automotive parts industry and therefore should be applied to completely-built units: The passage mentions that the deregulated automobile parts industry has shown better productivity and export performance, suggesting that deregulation could work for the completely-built units’ sector as well.

- (D) P and Q do not invest enough in research and development: The passage indicates that the automobile industry, dominated by P and Q, had very little research and development, implying that they do not invest enough in this area.


Step 3: Conclusion.

The correct inferences are (A), (C), and (D) based on the passage's explanation of the issues with the automobile industry and the better performance of the automobile parts sector.


Final Answer: (A), (C), (D)
Quick Tip: When analyzing an industry’s performance, factors such as tariff barriers and deregulation can significantly influence productivity and competitiveness.


Question 25:

According to the Path–Goal theory of leadership, a style of leadership in which the leader consults with subordinates, involving them in the decision-making process is ____________.

  • (A) participative
  • (B) directive
  • (C) supportive
  • (D) achievement oriented
Correct Answer: (A) participative
View Solution




Step 1: Understanding Path–Goal theory.

The Path–Goal theory explains how leaders motivate subordinates by clarifying goals and paths to achieve them. It identifies four leadership styles: directive, supportive, participative, and achievement-oriented.


Step 2: Analyzing the description.

The description emphasizes consultation with subordinates and their involvement in decision-making. This directly corresponds to the participative leadership style, where leaders seek input and share decision authority.


Step 3: Eliminating other options.

(B) Directive leadership involves giving clear instructions without consultation.

(C) Supportive leadership focuses on concern for subordinates’ well-being.

(D) Achievement-oriented leadership sets challenging goals and expects high performance.


Step 4: Conclusion.

Since the leader actively involves subordinates in decisions, the correct answer is participative leadership.



Final Answer: (A) participative
Quick Tip: Participative leadership emphasizes shared decision-making and employee involvement.


Question 26:

A process to test reliability that involves creating a large ‘bank’ of items and then dividing it into different versions or creating two tests with items of similar difficulty is called ________________________.

  • (A) alternate and parallel form reliability
  • (B) measure of test homogeneity
  • (C) test-retest
  • (D) inter-rater reliability
Correct Answer: (A) alternate and parallel form reliability
View Solution




Step 1: Understanding test reliability.

Reliability refers to the consistency of a measurement instrument. Different methods exist to assess reliability depending on how consistency is evaluated.


Step 2: Interpreting the description.

The process described involves constructing multiple versions of a test from a common item pool, ensuring similar difficulty levels across versions. This is characteristic of alternate or parallel form reliability.


Step 3: Eliminating incorrect options.

(B) Test homogeneity examines internal consistency, not multiple test forms.

(C) Test–retest reliability involves administering the same test at two different times.

(D) Inter-rater reliability concerns agreement between different evaluators.


Step 4: Conclusion.

The described method clearly matches alternate and parallel form reliability.



Final Answer: (A) alternate and parallel form reliability
Quick Tip: Parallel forms reliability checks consistency across equivalent versions of a test.


Question 27:

Delusion of ‘reference’ is a symptom of schizophrenia in which people ________________________.

  • (A) believe that random events or comments are directed at them
  • (B) believe that their thoughts are being broadcasted
  • (C) believe they have special powers
  • (D) have sensory experiences in the absence of stimulus
Correct Answer: (A)
View Solution




Step 1: Understanding delusion of reference.

A delusion of reference occurs when an individual interprets neutral or unrelated events as having special personal significance.


Step 2: Analysis of options.

(A) Correct: Believing that random events, remarks, or media messages are specifically directed at oneself defines delusion of reference.

(B) Refers to thought broadcasting, a different type of delusion.

(C) Describes grandiose delusions.

(D) Refers to hallucinations, not delusions.


Step 3: Conclusion.

The defining feature of delusion of reference is the personal misinterpretation of neutral events.



Final Answer: (A)
Quick Tip: Delusions of reference involve misattributing personal meaning to neutral events.


Question 28:

A student who has prepared inadequately for an exam attributes his failing grade to an unfair test, cheating by other students or a professor who taught badly, is expressing \hspace{2cm}.

  • (A) projection
  • (B) reaction formation
  • (C) repression
  • (D) rationalization
Correct Answer: (A)
View Solution




Step 1: Understanding the behaviour described.

The student fails to accept responsibility for inadequate preparation and instead blames external factors such as the examiner, other students, or the teacher.


Step 2: Explaining projection.

Projection is a defense mechanism in which an individual attributes their own faults, failures, or unacceptable feelings to others.


Step 3: Eliminating other options.

Reaction formation involves expressing opposite feelings, repression involves unconscious forgetting, and rationalization involves giving socially acceptable reasons rather than directly blaming others.


Step 4: Conclusion.

The student is projecting personal failure onto external agents.



Final Answer: (A)
Quick Tip: Projection helps individuals protect self-esteem by shifting blame onto others.


Question 29:

Which of the following theories of emotion best fits with the statement that perception of an environmental situation results in emotions and bodily reactions simultaneously, and that felt emotion and bodily reactions are independent of each other?

  • (A) Cannon-Bard Theory
  • (B) Lazarus’s Cognitive Appraisal Theory
  • (C) Schachter-Singer Theory
  • (D) James-Lange Theory
Correct Answer: (A)
View Solution




Step 1: Understanding the key claim.

The question states that emotional experience and bodily reactions occur simultaneously and independently after perceiving a stimulus.


Step 2: Explaining Cannon-Bard Theory.

According to the Cannon-Bard Theory, emotional experience and physiological arousal occur at the same time but are not causally dependent on each other.


Step 3: Eliminating other theories.

James-Lange Theory claims emotion follows bodily reactions, while Schachter-Singer and Lazarus emphasize cognitive appraisal as central to emotion.


Step 4: Conclusion.

The description matches the Cannon-Bard Theory.



Final Answer: (A)
Quick Tip: Cannon-Bard theory challenges the idea that emotions arise from bodily changes.


Question 30:

The perceived fairness of the distribution of resources and rewards is described as \hspace{2cm}.

  • (A) distributive justice
  • (B) procedural justice
  • (C) interactional justice
  • (D) informational justice
Correct Answer: (A)
View Solution




Step 1: Understanding the concept of fairness.

The question focuses on fairness related specifically to outcomes such as pay, rewards, or resources.


Step 2: Explaining distributive justice.

Distributive justice refers to perceived fairness in the allocation and distribution of outcomes among individuals.


Step 3: Distinguishing from other types of justice.

Procedural justice concerns fairness of decision-making processes, interactional justice concerns interpersonal treatment, and informational justice concerns adequacy of explanations.


Step 4: Conclusion.

Fairness in distribution is correctly described as distributive justice.



Final Answer: (A)
Quick Tip: Distributive justice answers the question: “Who gets what, and is it fair?”


Question 31:

According to Bandura’s theory of personality, which one of the following is the most important \textbf{person variables} in determining personality?

  • (A) Self-efficacy
  • (B) Self-concept
  • (C) Self-esteem
  • (D) Self-determination
Correct Answer: (A) Self-efficacy
View Solution




Step 1: Understanding Bandura’s social-cognitive theory.

Albert Bandura emphasized the role of cognitive processes in personality development, especially how individuals perceive their own abilities to act effectively in the world


Step 2: Defining self-efficacy.

Self-efficacy refers to a person’s belief in their capability to organize and execute actions required to manage prospective situationsThis belief strongly influences motivation, behavior, and emotional reactions


Step 3: Evaluating other options.

While self-concept and self-esteem are important aspects of personality, Bandura considered them less central than self-efficacySelf-determination is associated more with motivation theories than with Bandura’s core framework


Step 4: Final conclusion.

Thus, according to Bandura, self-efficacy is the most important person variable in determining personality
Quick Tip: In Bandura’s theory, what people believe they can do matters more than what they objectively can do


Question 32:

Match the events in the first column with the different categories of stress in the second column.



\begin{tabular{ l p{7cm l l
& Events & & Stress

P & A young child who loses his water bottle on the school bus & (i) & Traumatic event

Q & A person who escaped from the Taj hotel on 26th November, 2008 around midnight and has nightmares about the same incident & (ii) & Chronic stressor

R & Someone fearing the loss of one’s job & (iii) & Major life event

S & A woman who has her first baby & (iv) & Daily hassle

\end{tabular

  • (A) P-(iv), Q-(i), R-(ii), S-(iii)
  • (B) P-(iv), Q-(iii), R-(i), S-(ii)
  • (C) P-(ii), Q-(i), R-(iii), S-(iv)
  • (D) P-(i), Q-(iv), R-(ii), S-(iii)
Correct Answer: (A) P-(iv), Q-(i), R-(ii), S-(iii)
View Solution




Step 1: Identifying daily hassles.

Losing a water bottle is a minor, everyday inconvenienceThis fits the category of a daily hassleTherefore, P corresponds to (iv)


Step 2: Identifying traumatic events.

Surviving a terrorist attack and experiencing recurring nightmares is characteristic of traumaHence, Q corresponds to (i)


Step 3: Identifying chronic stressors.

Fear of job loss involves prolonged uncertainty and ongoing stressThis makes R a chronic stressor, corresponding to (ii)


Step 4: Identifying major life events.

The birth of a first child is a significant life transition involving major psychological and social changesThus, S corresponds to (iii)


Step 5: Final conclusion.

The correct matching is P-(iv), Q-(i), R-(ii), and S-(iii)
Quick Tip: Stressors can be classified by intensity and duration: daily hassles, chronic stressors, major life events, and traumatic events


Question 33:

Which of the following types of colour blindness denotes blue–yellow colour deficiency?

  • (A) Tritanopia
  • (B) Protanopia
  • (C) Deuteranopia
  • (D) Ritalin
Correct Answer: (A) Tritanopia
View Solution




Step 1: Understanding colour vision deficiencies.

Colour blindness results from the malfunctioning or absence of certain cone cells in the retina.


Step 2: Identifying Tritanopia.

Tritanopia is caused by defects in the blue-sensitive (S) cones and leads to difficulty distinguishing between blue and yellow colours.


Step 3: Elimination of other options.

Protanopia and Deuteranopia affect red–green colour perception, while Ritalin is a medication and unrelated to vision.


Step 4: Conclusion.

Hence, blue–yellow colour deficiency is known as Tritanopia.
Quick Tip: Tritanopia is rarer than red–green colour blindness.


Question 34:

Which of the following properties of sound is similar to the hue of light?

  • (A) Pitch
  • (B) Timbre
  • (C) Loudness
  • (D) Purity
Correct Answer: (A) Pitch
View Solution




Step 1: Understanding hue of light.

Hue refers to the quality of colour determined by the wavelength of light.


Step 2: Corresponding property in sound.

Pitch is determined by the frequency of a sound wave and plays a role analogous to wavelength in light.


Step 3: Eliminating other options.

Loudness corresponds to brightness, timbre to texture, and purity to spectral composition, not hue.


Step 4: Conclusion.

Therefore, pitch is the sound property analogous to the hue of light.
Quick Tip: Higher frequency means higher pitch, just as shorter wavelength means higher-energy colour.


Question 35:

\hspace{3cm} is a reinforcement schedule, where a person or animal receives the reinforcement based on varying amount of time.

  • (A) Variable-interval
  • (B) Fixed-ratio
  • (C) Fixed-interval
  • (D) Variable-ratio
Correct Answer: (A) Variable-interval
View Solution




Step 1: Understanding reinforcement schedules.

Reinforcement schedules describe how and when rewards are delivered following a behavior.


Step 2: Variable-interval schedule.

In a variable-interval schedule, reinforcement is given after unpredictable or varying time intervals, regardless of the number of responses.


Step 3: Distinction from other schedules.

Fixed schedules use constant time or response counts, while ratio schedules depend on number of responses rather than time.


Step 4: Conclusion.

Thus, reinforcement based on varying amounts of time is called a variable-interval schedule.
Quick Tip: Checking emails is a classic example of a variable-interval reinforcement schedule.


Question 36:

\hspace{3cm} is a loss of memory of events that occurred prior to the trauma.

  • (A) Retrograde amnesia
  • (B) Anterograde amnesia
  • (C) Infantile amnesia
  • (D) Posthypnotic amnesia
Correct Answer: (A)
View Solution




Step 1: Understand types of amnesia.

Retrograde amnesia refers to the inability to recall information or events that occurred before a traumatic incident.


Step 2: Eliminate incorrect options.

Anterograde amnesia affects the formation of new memories after trauma.

Infantile amnesia refers to the inability to recall early childhood memories.

Posthypnotic amnesia involves memory loss following hypnosis.


Step 3: Conclusion.

Therefore, the correct answer is Retrograde amnesia.
Quick Tip: Retrograde = backward loss of memory; Anterograde = forward loss of memory.


Question 37:

Phobias and Obsessive Compulsive Disorder fall in the category of \hspace{3cm}.

  • (A) anxiety disorders
  • (B) mood disorders
  • (C) somatoform disorders
  • (D) psychotic disorders
Correct Answer: (A)
View Solution




Step 1: Identify the nature of phobias and OCD.

Both phobias and Obsessive Compulsive Disorder are characterized by excessive fear, anxiety, and avoidance or compulsive behaviors.


Step 2: Match with diagnostic categories.

These features place both conditions within the category of anxiety disorders.


Step 3: Eliminate incorrect options.

Mood disorders primarily involve disturbances of mood such as depression or mania.

Somatoform disorders involve physical symptoms without medical cause.

Psychotic disorders involve loss of contact with reality.


Step 4: Conclusion.

Hence, the correct answer is anxiety disorders.
Quick Tip: OCD and phobias are traditionally grouped under anxiety disorders in psychological classification.


Question 38:

The smallest unit of speech perception that has meaning is \hspace{3cm}.

  • (A) morpheme
  • (B) syntax
  • (C) semantics
  • (D) phoneme
Correct Answer: (A)
View Solution




Step 1: Define linguistic units.

A morpheme is the smallest linguistic unit that carries meaning, such as roots, prefixes, or suffixes.


Step 2: Distinguish from phonemes.

Phonemes are the smallest units of sound but do not carry meaning on their own.


Step 3: Eliminate incorrect options.

Syntax and semantics are branches of linguistics, not units of speech perception.


Step 4: Conclusion.

Therefore, the smallest meaningful unit of speech perception is a morpheme.
Quick Tip: Meaning begins at the morpheme level, not at the phoneme level.


Question 39:

In adolescence, with the development of the stage of ‘formal operations’, we are likely to see the development of ________________________.

  • (A) post-conventional morality
  • (B) pre-conventional morality
  • (C) bodily-kinesthetic intelligence
  • (D) transference
Correct Answer: (A) post-conventional morality
View Solution




Step 1: Understanding formal operations.

According to Jean Piaget, the stage of formal operations develops during adolescence and is characterized by abstract, hypothetical, and logical thinking.


Step 2: Linking cognitive and moral development.

Lawrence Kohlberg proposed that higher levels of moral reasoning depend on advanced cognitive abilities. The capacity for abstract reasoning enables individuals to think beyond social rules and authority.


Step 3: Analysis of options.

(A) Post-conventional morality: Correct, as this stage involves reasoning based on universal ethical principles, which requires abstract thought.

(B) Pre-conventional morality is typical of early childhood.

(C) Bodily-kinesthetic intelligence is unrelated to Piaget’s cognitive stages.

(D) Transference is a psychoanalytic concept, not a developmental stage.


Step 4: Conclusion.

The emergence of formal operational thinking supports the development of post-conventional moral reasoning.



Final Answer: (A) post-conventional morality
Quick Tip: Advanced moral reasoning requires abstract thinking, which develops during Piaget’s formal operational stage.


Question 40:

Which technique allows researchers to conduct an integrative statistical analysis of multiple independent studies addressing the same question?

  • (A) Meta-analysis
  • (B) Correlational analysis
  • (C) Regression analysis
  • (D) Bootstrapping
Correct Answer: (A) Meta-analysis
View Solution




Step 1: Understanding the research context.

Researchers often need to synthesize findings from multiple independent studies to draw stronger and more reliable conclusions.


Step 2: Defining meta-analysis.

Meta-analysis is a statistical technique that combines results from several studies on the same topic to estimate an overall effect size.


Step 3: Eliminating incorrect options.

(B) Correlational analysis examines relationships within a single dataset.

(C) Regression analysis predicts relationships among variables within one study.

(D) Bootstrapping is a resampling technique, not a method for combining multiple studies.


Step 4: Conclusion.

Only meta-analysis allows systematic and integrative analysis across multiple independent studies.



Final Answer: (A) Meta-analysis
Quick Tip: Meta-analysis increases statistical power by pooling data across studies.


Question 41:

The role of culture and ‘scaffolding’ are emphasized in __________.

  • (A) Vygotsky’s theory of cognitive development
  • (B) Piaget’s theory of cognitive development
  • (C) Atkinson-Shiffrin’s information-processing model
  • (D) Karmiloff-Smith’s theory of cognitive development
Correct Answer: (A) Vygotsky’s theory of cognitive development
View Solution




Step 1: Understanding scaffolding.

Scaffolding refers to the support provided by more knowledgeable others to help learners achieve tasks they cannot complete independently.


Step 2: Cultural emphasis in development.

Lev Vygotsky emphasized that cognitive development is socially and culturally mediated, particularly through language and interaction.


Step 3: Analysis of options.

(A) Correct: Vygotsky introduced concepts like the Zone of Proximal Development (ZPD) and scaffolding.

(B) Piaget focused on individual discovery rather than social mediation.

(C) Atkinson-Shiffrin deals with memory systems, not social learning.

(D) Karmiloff-Smith focuses on representational redescription, not scaffolding.


Step 4: Conclusion.

Culture and guided interaction are central to Vygotsky’s theory of cognitive development.



Final Answer: (A) Vygotsky’s theory of cognitive development
Quick Tip: Scaffolding and ZPD are hallmark concepts of Vygotsky’s socio-cultural theory.


Question 42:

Most people tend NOT to consider situational factors while judging others’ behaviour because \hspace{2.5cm}.

  • (A) people are inclined to commit the fundamental attribution error
  • (B) of the frustration-aggression relationship
  • (C) people are influenced by the laws of reinforcement
  • (D) of the overjustification effect
Correct Answer: (A)
View Solution




Step 1: Understanding the judgment error.

When judging others’ behaviour, people often overemphasize personal traits and underemphasize situational influences.


Step 2: Explaining the fundamental attribution error.

The fundamental attribution error refers to the tendency to attribute others’ actions to internal dispositions rather than external circumstances.


Step 3: Eliminating other options.

The frustration-aggression hypothesis explains aggression, reinforcement laws explain learning, and the overjustification effect concerns motivation, not social judgment.


Step 4: Conclusion.

Hence, people neglect situational factors because they commit the fundamental attribution error.



Final Answer: (A)
Quick Tip: When judging others, people focus more on “who they are” than on “what situation they are in.”


Question 43:

Using archival analysis, scientists describe a culture by \hspace{3cm}.

  • (A) examining documents like magazines, diaries and newspapers
  • (B) surveying a representative sample of members of the society
  • (C) observing the behaviour of members of the society
  • (D) comparing the direct observations of behaviour from different cultures
Correct Answer: (A)
View Solution




Step 1: Understanding archival analysis.

Archival analysis involves the systematic study of existing records and documents rather than direct interaction with participants.


Step 2: Identifying the correct method.

Materials such as newspapers, magazines, letters, and diaries provide historical and cultural data that researchers analyze to understand social patterns.


Step 3: Eliminating incorrect options.

Surveying and observing involve direct data collection, while cross-cultural comparison relies on observational methods rather than archival records.


Step 4: Conclusion.

Thus, archival analysis describes culture through the examination of existing documents.



Final Answer: (A)
Quick Tip: Archival research studies culture through records created independently of the researcher.


Question 44:

Which is/are the component(s) of Gardner’s theory of multiple intelligences?

  • (A) Logical-Mathematical intelligence
  • (B) Linguistic intelligence
  • (C) Spatial intelligence
  • (D) Insight
Correct Answer: (A), (B), and (C)
View Solution




Step 1: Understanding Gardner’s theory.

Howard Gardner proposed the theory of multiple intelligences to challenge the traditional view of intelligence as a single, general abilityHe argued that intelligence is better understood as a set of distinct capacities


Step 2: Identifying core intelligences.

Among the intelligences identified by Gardner are Logical–Mathematical intelligence, Linguistic intelligence, and Spatial intelligenceThese are central components of his framework and are well-established in educational psychology


Step 3: Evaluating option (D).

Insight is not listed as a separate intelligence in Gardner’s theoryIt may relate to problem-solving or creativity but does not constitute an independent intelligence category


Step 4: Final conclusion.

Therefore, the correct components of Gardner’s theory of multiple intelligences are (A), (B), and (C)
Quick Tip: Gardner’s theory emphasizes diverse forms of intelligence beyond traditional IQ measures


Question 45:

Match the neurotransmitter in the first column with its effect in the second column.


  • (A) P-(iv), Q-(i), R-(ii), S-(iii)
  • (B) P-(iii), Q-(i), R-(ii), S-(v)
  • (C) P-(i), Q-(ii), R-(iv), S-(iii)
  • (D) P-(iv), Q-(i), R-(v), S-(iii)
Correct Answer: (A) P-(iv), Q-(i), R-(ii), S-(iii)
View Solution




Step 1: Acetylcholine (P).

Acetylcholine is involved in muscle contraction at neuromuscular junctions and also plays a crucial role in learning and memory. Hence, P matches with (iv).


Step 2: Norepinephrine (Q).

Norepinephrine is primarily associated with alertness, arousal, and wakefulness, especially in the sympathetic nervous system. Thus, Q matches with (i).


Step 3: Serotonin (R).

Serotonin regulates mood, appetite, sleep, and emotional states. Therefore, R corresponds to (ii).


Step 4: GABA (S).

Gamma-aminobutyric acid (GABA) is the major inhibitory neurotransmitter in the brain, reducing neuronal excitability. Hence, S matches with (iii).


Step 5: Conclusion.

The correct matching sequence is P-(iv), Q-(i), R-(ii), and S-(iii), which corresponds to option (A).
Quick Tip: GABA inhibits neural activity, while acetylcholine excites muscle action and cognitive functions.


Question 46:

Match the depth cues in the first column with their description in the second column.



Cues \hspace{5cm} Description


P. Aerial perspective \hspace{1.6cm
(i) The inward turning of the eyes that occurs when you look at an object that is closer to you.


Q. Motion Parallax \hspace{1.7cm
(ii) The distant mountain appears fuzzy.


R. Convergence \hspace{2.4cm
(iii) Parallel lines appear to converge at the horizon.


S. Linear perspective \hspace{1.3cm
(iv) When we travel in a vehicle, objects that are close appear to move in the opposite direction.

  • (A) P-(ii), Q-(iv), R-(i), S-(iii)
  • (B) P-(ii), Q-(iii), R-(iv), S-(i)
  • (C) P-(iv), Q-(i), R-(ii), S-(iii)
  • (D) P-(iii), Q-(iv), R-(i), S-(ii)
Correct Answer: (A)
View Solution




Step 1: Identify Aerial Perspective (P).

Aerial perspective refers to the effect of the atmosphere on distant objects, making them appear hazy or fuzzy.
This matches description (ii): “The distant mountain appears fuzzy.”


Step 2: Identify Motion Parallax (Q).

Motion parallax occurs when objects closer to the observer appear to move faster and in the opposite direction compared to distant objects when the observer is moving.
This corresponds to description (iv).


Step 3: Identify Convergence (R).

Convergence is the inward turning of the eyes when focusing on nearby objects.
This matches description (i).


Step 4: Identify Linear Perspective (S).

Linear perspective is the visual cue where parallel lines appear to meet at a point on the horizon.
This corresponds to description (iii).


Step 5: Conclusion.

Thus, the correct matching is \[ P-(ii), Q-(iv), R-(i), S-(iii). \]
Quick Tip: Depth perception relies on multiple visual cues—some depend on eye movement (convergence), while others depend on motion or atmospheric effects.


Question 47:

Match the concepts in the first column with the description in the second column.



\begin{tabular{|c|l|c|l|
\hline
& Concept & & Description

\hline
P & Transience & (i) & Source of memory is confused

Q & Misattribution & (ii) & Inability to forget undesirable memories

R & Absentmindedness & (iii) & Accessibility of memory decreases over time

S & Persistence & (iv) & Forgetting caused by lapses in attention

\hline
\end{tabular

  • (A) P-(iii), Q-(i), R-(iv), S-(ii)
  • (B) P-(i), Q-(ii), R-(iii), S-(iv)
  • (C) P-(ii), Q-(i), R-(iv), S-(iii)
  • (D) P-(iii), Q-(iv), R-(i), S-(ii)
Correct Answer: (A)
View Solution




Step 1: Recall the “seven sins of memory” (Schacter).

These concepts describe characteristic ways in which memory can fail or distort.


Step 2: Match each concept with its description.

Transience refers to the weakening of memory traces over time, hence it matches with (iii) accessibility of memory decreases over time.

Misattribution involves remembering content but confusing its source, so it matches with (i) source of memory is confused.

Absentmindedness is forgetting due to failures of attention at encoding or retrieval, which matches (iv) forgetting caused by lapses in attention.

Persistence refers to the unwanted recurrence of memories, matching (ii) inability to forget undesirable memories.


Step 3: Identify the correct option.

The correct mapping is:

P-(iii), Q-(i), R-(iv), S-(ii), which corresponds to option (A).



Final Answer: (A)
Quick Tip: Transience fades memories, misattribution mixes sources, absentmindedness reflects attention failures, and persistence keeps unwanted memories alive.


Question 48:

Which of the following will hold true for a learning acquisition curve drawn for a classical conditioning experiment on eye blinking as conditioned response that plots the learning over a number of trials?

  • (A) The rate of learning on earlier trials is more than that on later trials.
  • (B) The learning curve is negatively accelerated.
  • (C) The rate of learning in later trials will be more than that on earlier trials.
  • (D) The rate of learning is proportionally increasing with increasing number of trials.
Correct Answer: (A) and (B)
View Solution




Step 1: Understanding learning acquisition curves.

In classical conditioning, learning typically occurs rapidly during the initial trials and then slows down as performance approaches an asymptote.


Step 2: Evaluating option (A).

The rate of learning is higher during early trials because the organism is forming new associations quickly. Hence, learning is faster initially than in later trials.


Step 3: Evaluating option (B).

Such learning curves are described as negatively accelerated because the slope is steep at the beginning and gradually flattens over time.


Step 4: Eliminating incorrect options.

Options (C) and (D) incorrectly suggest that learning increases with more trials, which contradicts established findings in conditioning research.


Step 5: Conclusion.

Therefore, options (A) and (B) correctly describe the learning acquisition curve.



Final Answer: (A) and (B)
Quick Tip: Most learning curves show rapid initial gains followed by gradual improvement.


Question 49:

In the situation where ‘a teenager who hates studying science but is also not able to tell his parents fearing their reaction’, what is the conflict he is facing and which of these may be his way of dealing with this situation?

  • (A) Avoidance-Avoidance conflict, he will keep vacillating between telling his parents and continuing to study science.
  • (B) Avoidance-Avoidance conflict, he may contemplate running away from his home.
  • (C) Approach-Avoidance conflict, he will experience some emotional turmoil.
  • (D) Multiple Approach-Avoidance conflict, he will be guided by internal values.
Correct Answer: (A) and (B)
View Solution




Step 1: Identifying the nature of the conflict.

The teenager faces two undesirable options: continuing to study science (which he hates) or telling his parents and facing their negative reaction.


Step 2: Explaining avoidance-avoidance conflict.

An avoidance-avoidance conflict occurs when an individual must choose between two negative alternatives, both of which are unpleasant.


Step 3: Evaluating coping behaviours.

Such conflicts often lead to vacillation or escape-oriented responses, such as avoiding decision-making or contemplating running away.


Step 4: Eliminating incorrect options.

Approach-avoidance conflict involves a single goal with both positive and negative aspects, which is not the case here.


Step 5: Conclusion.

Hence, the correct conflict type and coping responses are described in options (A) and (B).



Final Answer: (A) and (B)
Quick Tip: Avoidance-avoidance conflicts often result in indecision or escape behaviours.


Question 50:

Which of these is true about individuals high on \textbf{n}-achievement motivation?

  • (A) High \textbf{n}-achievement individuals like to work on situations where they have control and can get feedback.
  • (B) High \textbf{n}-achievement individuals persistently work on tasks they perceive as either reflecting their personal characteristics like intelligence or are career-related.
  • (C) High \textbf{n}-achievement individuals prefer working on extremely challenging tasks for bigger gains.
  • (D) High \textbf{n}-achievement individuals avoid changing their aspiration levels.
Correct Answer: (A) and (B)
View Solution




Step 1: Understanding \textbf{n}-achievement motivation.

According to McClelland, individuals high in achievement motivation prefer tasks where outcomes depend on their own effort and where performance feedback is available


Step 2: Evaluating option (A).

High n-achievement individuals prefer situations with personal control and clear feedbackThis directly matches McClelland’s descriptionHence, (A) is correct


Step 3: Evaluating option (B).

Such individuals persist on tasks that reflect their competence and are linked to achievement and career successTherefore, (B) is also correct


Step 4: Eliminating options (C) and (D).

High n-achievement individuals prefer moderately challenging tasks, not extremely difficult onesThey also adjust aspiration levels based on feedbackThus, (C) and (D) are incorrect


Step 5: Final conclusion.

The correct statements are (A) and (B)
Quick Tip: Achievement-motivated individuals seek moderate risk, responsibility, and performance feedback


Question 51:

Which of the following is/are Allport’s basic assumption(s) concerning human nature?

  • (A) Human growth as an active process of “becoming”.
  • (B) Personality cannot be fully understood by examining each trait separately, though some system of conceptual schemata are essential for personality study and a trait must be related to the total pattern of personality.
  • (C) Personality is organized in a \textbf{topographical model}.
  • (D) All human events are determined by powerful instinctual forces.
Correct Answer: (A) and (B)
View Solution




Step 1: Allport’s view of human nature.

Gordon Allport viewed human beings as proactive, forward-looking, and capable of growthHe emphasized individuality and conscious motivation


Step 2: Evaluating option (A).

Allport described personality as a dynamic process of “becoming,” highlighting growth and self-directionThus, (A) is correct


Step 3: Evaluating option (B).

Allport stressed that traits must be understood within the context of the whole personality, not in isolationHence, (B) is correct


Step 4: Eliminating options (C) and (D).

The topographical model and instinctual determinism are associated with Freud, not AllportTherefore, (C) and (D) are incorrect


Step 5: Final conclusion.

The correct assumptions are (A) and (B)
Quick Tip: Allport emphasized conscious motivation and personal growth rather than unconscious drives


Question 52:

Which of the following aspects are characteristics of the group structure in an organizational context?

  • (A) Task-oriented role (the activities of an individual that involve helping the group reach the goal).
  • (B) Socio-emotional role (the activities of an individual that involve being supportive and nurturing of other group members).
  • (C) Prescriptive norms (expectations within groups regarding what has to be done).
  • (D) Monitoring (observing work performance).
Correct Answer: (A), (B), and (C)
View Solution




Step 1: Understanding group structure.

Group structure in organizations includes roles, norms, and patterns of interaction that guide member behavior


Step 2: Evaluating option (A).

Task-oriented roles focus on goal accomplishment and are a core component of group structureThus, (A) is correct


Step 3: Evaluating option (B).

Socio-emotional roles maintain harmony and support among membersThese roles are essential to group functioningHence, (B) is correct


Step 4: Evaluating option (C).

Prescriptive norms define expected behaviors within the group and are central to group structureTherefore, (C) is correct


Step 5: Eliminating option (D).

Monitoring refers more to managerial control processes rather than structural characteristics of groupsHence, (D) is incorrect


Step 6: Final conclusion.

The characteristics of group structure are (A), (B), and (C)
Quick Tip: Effective groups balance task roles, socio-emotional roles, and clear norms


Question 53:

Which of the following needs/motives are proposed by Abraham Maslow?

  • (A) Deficit needs.
  • (B) Metaneeds.
  • (C) Self-actualization needs.
  • (D) Need-achievement.
Correct Answer: (A), (B), (C)
View Solution




Step 1: Maslow’s hierarchy of needs.

Abraham Maslow proposed a hierarchical model of human motivation, beginning with basic needs and culminating in higher psychological growth needs.


Step 2: Deficit needs.

Deficit needs include physiological, safety, love/belonging, and esteem needs. Their absence creates motivation, making option (A) correct.


Step 3: Self-actualization and metaneeds.

Self-actualization represents the fulfillment of one’s potential, while metaneeds (such as truth, beauty, and justice) emerge at higher levels of psychological development. Hence, options (B) and (C) are correct.


Step 4: Elimination of option (D).

Need for achievement was proposed by McClelland, not Maslow.


Step 5: Conclusion.

Therefore, options (A), (B), and (C) are correct.
Quick Tip: Maslow distinguished between deficiency needs and growth-oriented metaneeds.


Question 54:

A researcher is conducting a study involving two independent variables. Which of the following Analysis of Variance (ANOVA) is/are available to him/her?

  • (A) \(2 \times 2\) ANOVA.
  • (B) \(4 \times 3\) ANOVA.
  • (C) \(3 \times 2 \times 2\) ANOVA.
  • (D) One-way ANOVA.
Correct Answer: (A), (B)
View Solution




Step 1: Understanding independent variables in ANOVA.

The number of independent variables determines the type of ANOVA used. Two independent variables require a two-way (factorial) ANOVA.


Step 2: Evaluation of options.

Options (A) and (B) both represent factorial ANOVAs with two independent variables, differing only in the number of levels.


Step 3: Elimination of incorrect options.

Option (C) involves three independent variables, while option (D) involves only one independent variable.


Step 4: Conclusion.

Hence, options (A) and (B) are correct.
Quick Tip: The number of factors in ANOVA equals the number of independent variables.


Question 55:

Which of the following statements is true about personal space?

  • (A) It is silent and invisible.
  • (B) It is a geographic component of interpersonal relations.
  • (C) Invasions of personal space are a matter of degree.
  • (D) There are no across culture variations.
Correct Answer: (A), (B), (C)
View Solution




Step 1: Understanding personal space.

Personal space refers to the invisible boundary individuals maintain around themselves in social interactions.


Step 2: Evaluation of correct statements.

Personal space is silent and invisible, making option (A) correct.

It functions as a spatial or geographic component in interpersonal relations, validating option (B).

Violations of personal space can vary in intensity and context, so option (C) is also correct.


Step 3: Elimination of option (D).

Personal space varies significantly across cultures, making option (D) incorrect.


Step 4: Conclusion.

Thus, options (A), (B), and (C) are correct.
Quick Tip: Personal space norms differ across cultures, contexts, and relationships.


Question 56:

If a person got a score of 75 on a test, which of the following distributions allow(s) for the most favourable interpretation of that score? (assuming higher values are more favourable)

  • (A) Mean = 55, Standard Deviation = 4
  • (B) Mean = 60, Standard Deviation = 3
  • (C) Mean = 65, Standard Deviation = 5
  • (D) Mean = 50, Standard Deviation = 10
Correct Answer: (A), (B)
View Solution




Step 1: Use standard score (z-score) for comparison.

A score is interpreted more favourably when it lies many standard deviations above the mean.


Step 2: Compute relative standing.

(A): \( z = \frac{75-55}{4} = 5 \)

(B): \( z = \frac{75-60}{3} = 5 \)

(C): \( z = \frac{75-65}{5} = 2 \)

(D): \( z = \frac{75-50}{10} = 2.5 \)


Step 3: Compare outcomes.

Options (A) and (B) yield the highest z-scores, hence the most favourable interpretations.


Step 4: Conclusion.

The most favourable interpretations occur in distributions (A) and (B).
Quick Tip: A higher z-score means better relative performance, regardless of the raw score.


Question 57:

A man’s wife is dying. She is in dire need of a drug. The only place to get the drug is at the store of a pharmacist who is known to overcharge people. The man can only pay ₹20000 but the pharmacist wants ₹50000. What will the man do if he is at the conventional stage of morality as per Kohlberg’s theory of moral reasoning?

  • (A) He would not steal the drug, as everyone will see him as a thief, and his wife would not approve of his stealing.
  • (B) No matter what, he would obey the law because stealing is a crime.
  • (C) The man would not steal the drug, as he may get caught and go to jail.
  • (D) The man would steal the drug to cure his wife and then inform the authorities.
Correct Answer: (A), (B)
View Solution




Step 1: Recall the conventional level.

The conventional stage emphasizes social approval, law, and order.


Step 2: Evaluate options.

(A) reflects concern for social approval and expectations.

(B) reflects respect for law and authority.

(C) reflects fear of punishment (pre-conventional).

(D) reflects principled reasoning (post-conventional).


Step 3: Conclusion.

Thus, responses (A) and (B) align with conventional morality.
Quick Tip: Conventional morality focuses on maintaining social order and approval.


Question 58:

According to Erik Erikson, identity versus role confusion is the fifth stage of life span development. Which of the following outcomes may emerge as a result of identity crisis?

  • (A) Identity diffusion
  • (B) Identity moratorium
  • (C) Identity foreclosure
  • (D) Identity perception
Correct Answer: (A), (B)
View Solution




Step 1: Understand identity outcomes.

Identity crisis can lead to different identity statuses described by Erikson and Marcia.


Step 2: Evaluate the options.

(A) Identity diffusion involves lack of commitment and exploration.

(B) Identity moratorium involves active exploration without commitment.

(C) Identity foreclosure involves commitment without exploration.

(D) Identity perception is not a recognized identity status.


Step 3: Conclusion.

Outcomes directly linked to identity crisis are (A) and (B).
Quick Tip: Identity moratorium reflects exploration; diffusion reflects confusion.


Question 59:

Life span experts argue that biological aging begins at birth. Which of the following is/are explanation/s of aging?

  • (A) Cells can divide to a maximum of about 75 to 80 times and that as people age, cells become less capable of dividing.
  • (B) People age because when cells metabolize energy, the by-products include unstable oxygen molecules known as free radicals.
  • (C) Aging is due to the decay of mitochondria.
  • (D) Cell division increases as people age.
Correct Answer: (A), (B), (C)
View Solution




Step 1: Understanding biological explanations of aging.

Biological theories of aging explain aging as a result of cellular and physiological processes that gradually reduce efficiency and repair mechanisms over time.


Step 2: Analysis of options.

(A) Correct: This refers to the Hayflick limit, which states that cells have a finite capacity to divide, leading to aging as division slows down.

(B) Correct: Free radical theory explains aging as damage caused by unstable oxygen molecules produced during metabolism.

(C) Correct: Mitochondrial decay theory suggests that damage to mitochondria impairs cellular energy production, contributing to aging.

(D) Incorrect: Cell division generally decreases, not increases, with age.


Step 3: Conclusion.

Valid biological explanations of aging include cellular division limits, free radical damage, and mitochondrial decay.



Final Answer: (A), (B), (C)
Quick Tip: Most biological theories of aging focus on cumulative cellular damage and reduced regenerative capacity.


Question 60:

Which of the following are involved in sympathetic nervous system activation?

  • (A) Inhibited salivation
  • (B) Increased heart rate
  • (C) Inhibited digestion
  • (D) Constricted pupils
Correct Answer: (A), (B), (C)
View Solution




Step 1: Understanding the sympathetic nervous system.

The sympathetic nervous system prepares the body for “fight or flight” responses during stress or emergencies.


Step 2: Analysis of options.

(A) Correct: Salivation is reduced as bodily resources are redirected.

(B) Correct: Heart rate increases to supply more oxygen to muscles.

(C) Correct: Digestive activity is inhibited during sympathetic activation.

(D) Incorrect: The sympathetic system causes pupil dilation, not constriction.


Step 3: Conclusion.

The correct physiological responses associated with sympathetic activation are (A), (B), and (C).



Final Answer: (A), (B), (C)
Quick Tip: Remember: sympathetic activation speeds up the body, parasympathetic calms it down.


Question 61:

How is/are genetic theories of psychopathology tested?

  • (A) Using twin studies
  • (B) Using family history studies
  • (C) Using adoption studies
  • (D) Using psychodynamic studies
Correct Answer: (A), (B), (C)
View Solution




Step 1: Understanding genetic theories of psychopathology.

Genetic theories propose that mental disorders have a hereditary component, which can be tested by examining patterns of inheritance.


Step 2: Analysis of methods.

(A) Correct: Twin studies compare concordance rates between identical and fraternal twins.

(B) Correct: Family studies examine the prevalence of disorders among biological relatives.

(C) Correct: Adoption studies separate genetic influence from environmental factors by studying adopted individuals.

(D) Incorrect: Psychodynamic studies focus on unconscious processes, not genetic inheritance.


Step 3: Conclusion.

Genetic theories are empirically tested using twin, family, and adoption studies.



Final Answer: (A), (B), (C)
Quick Tip: Twin, family, and adoption studies are key tools to disentangle genetic and environmental influences.


Question 62:

An investigator approached college students who initially believed that water should be purified and asked them to compose and recite a videotaped speech against the use of water purifiers. To do this, some were offered large incentives and others were offered small incentives. Later their attitudes towards water purifiers were tested. Which of the following will be the expected finding(s)?

  • (A) The smaller the incentive, the greater will be the attitude change.
  • (B) All the students will change their attitudes towards water purifiers.
  • (C) All the students will continue to feel favourably about water purifiers.
  • (D) The larger the incentive, the greater will be the attitude change.
Correct Answer: (A)
View Solution




Step 1: Identifying the psychological principle.

This situation illustrates cognitive dissonance, where individuals experience discomfort when their actions conflict with their prior attitudes.


Step 2: Role of incentive size.

When incentives are small, individuals lack sufficient external justification for their counter-attitudinal behaviour and therefore change their internal attitudes to reduce dissonance.


Step 3: Eliminating incorrect options.

Large incentives provide adequate external justification, leading to less attitude change. Hence, options (B), (C), and (D) are incorrect.


Step 4: Conclusion.

Thus, smaller incentives produce greater attitude change.



Final Answer: (A)
Quick Tip: Less external justification often leads to greater internal attitude change due to cognitive dissonance.


Question 63:

In which attachment style(s), do children tend to show clingy behaviour but then reject the attachment figure’s attempt to interact with them?

  • (A) Resistant attachment
  • (B) Avoidant attachment
  • (C) Secure attachment
  • (D) Disorganized attachment
Correct Answer: (A)
View Solution




Step 1: Understanding attachment styles.

Attachment styles describe characteristic patterns of emotional bonds formed between infants and caregivers.


Step 2: Explaining resistant attachment.

Children with resistant (ambivalent) attachment show clingy behaviour but also resist or reject comfort from the caregiver upon reunion.


Step 3: Eliminating other options.

Avoidant attachment involves emotional distancing, secure attachment shows comfort-seeking and ease of interaction, and disorganized attachment involves inconsistent or confused behaviours.


Step 4: Conclusion.

Therefore, resistant attachment best fits the described behaviour.



Final Answer: (A)
Quick Tip: Resistant attachment combines proximity-seeking with anger or resistance toward the caregiver.


Question 64:

A study is conducted to see the effectiveness of volume levels of commercials. In the graph given below, values of mean effectiveness (dependent variable) are presented on the Y axis, and three levels of volume, namely, soft, medium and loud are presented on the X axis. The graph shows the relationship between volume and mean effectiveness for males in the continuous line and for females in the dashed line. Which of the following interpretation(s) is/are correct from the graph?

  • (A) A linear relationship exists for males and a nonlinear relationship for females.
  • (B) The effect of increasing volume on effectiveness depends on whether the participant is a male or a female.
  • (C) No interaction effect is seen.
  • (D) A linear relationship exists for both males and females.
Correct Answer: (A) and (B)
View Solution




Step 1: Examining the pattern for males.

The solid line representing males shows a steady increase in mean effectiveness from soft to medium to loud volumeThis consistent upward trend indicates a linear relationship between volume and effectiveness for males


Step 2: Examining the pattern for females.

The dashed line representing females increases from soft to medium volume but then decreases from medium to loud volumeThis rise-and-fall pattern indicates a nonlinear relationship between volume and effectiveness for females


Step 3: Assessing interaction effects.

The lines for males and females are not parallelThe effect of volume on effectiveness differs across genderFor example, loud volume increases effectiveness for males but decreases it for femalesThis indicates an interaction between volume level and gender


Step 4: Evaluating the options.

Option (A) is correct because males show a linear trend while females show a nonlinear trend

Option (B) is correct because the effect of volume clearly depends on gender

Option (C) is incorrect because a clear interaction effect is present

Option (D) is incorrect because females do not show a linear relationship


Step 5: Final conclusion.

The correct interpretations from the graph are (A) and (B)
Quick Tip: In interaction plots, non-parallel lines indicate an interaction effect between independent variables



Quick Links:

GATE 2021 XH Detailed Paper Analysis

The paper was divided into three parts. The first part was dedicated to the General Aptitude. The second and third part were dedicated to reasoning and comprehension, and optional subject respectively

  • The General Aptitude section carried 15 marks, distributed between 10 MCQs (each 5 questions carrying 1 mark and 2 marks respectively)
  • The reasoning and comprehension section (XH-B1) carried 25 marks, comprises of 15 questions, 10 MCQs and 5 MSQs
  • The MCQs carried 15 marks (5 questions carrying 1 mark and 5 questions carrying 2 marks)
  • The MSQs carried 10 marks, distributed between 5 questions (2 marks each question)
  • The remaining 40 questions, carrying 60 marks were related to the core optional subject

GATE 2021 XH Exam Pattern and Marking Scheme

  • Candidates would require to solve 65 questions in 3 hours in GATE 2021 XH
  • All the papers comprises of 65 questions (10 from General Aptitude and the other 55 from the optional subject)
  • Every incorrect MCQ will cost 1 mark reduction out of the final score
  • ⅓ marks would be deducted for every wrong MCQ carrying 1 mark, and ⅔ marks would be deducted for every wrong MCQ carrying 2 marks
  • NATs (Numerical Answer Types) and MSQs (Multiple Select Questions) don't follow any negative marking scheme

Also Check:

GATE Previous Year Question Papers:

Other PG Exam Question Papers:

0

Fees Structure

Structure based on different categories

CategoriesState
General2000
Women1000
sc1000
pwd1000

Note: GATE 2026 Application Fee must be paid online through net banking, debit card, or credit card facilities.

In case of any inaccuracy, Notify Us! 

Comments


No Comments To Show